AMC 8 Daily Practice Round 11

Complete problem set with solutions and individual problem pages

Problem 1 Easy

(888+777)\div (666+555+444)=            .

  • A.

    1

  • B.

    2

  • C.

    3

  • D.

    4

  • E.

    5

Answer:A

Dividing both the numerator and the denominator by 111, we get:

(8+7) \div (6+5+4) = 1.

Thus, the correct answer is A.

Link Problem
Problem 2 Medium

As shown in the figure, the area of the unshaded region is 15 square meters larger than the area of the shaded region. What is the area of the shaded region (in square meters)?

  • A.

    75

  • B.

    37.5

  • C.

    42

  • D.

    50

  • E.

    60

Answer:B

The ratio of the areas of the unshaded region to the shaded region is (12+2):10 = 7:5. Since the difference in their areas is 15 square meters, the area of the shaded region is calculated as:

\frac{15}{2} \times 5 = 37.5 \ \text{square meters}.

Link Problem
Problem 3 Medium

A shipping company charges the following rates: for a package weighing up to 5 kilograms, the cost is 13 dollars. For every kilogram exceeding 5 kilograms, an additional 2 dollars is charged per kilogram. If Emma ships a package weighing 8 kilograms, how much in dollars does she need to pay?

  • A.

    17

  • B.

    19

  • C.

    21

  • D.

    23

  • E.

    25

Answer:B

13+\left( 8-5 \right)\times 2=13+6=19.

Link Problem
Problem 4 Easy

Dan bought 4 books for the classroom library. The most expensive book costs \$26, and the least expensive book costs \$22. Based on this, which of the following cannot be the total cost of the 4 books?

  • A.

    \$ 88

  • B.

    \$ 92

  • C.

    \$ 94

  • D.

    \$ 98

  • E.

    \$ 100

Answer:A

The minimum total cost is calculated by taking 3 of the least expensive items and 1 of the most expensive item:

22 \times 3 + 26 = \$ 92 .

The maximum total cost is calculated by taking 3 of the most expensive items and 1 of the least expensive item 26 \times 3 + 22 = \$ 100.

Thus, the total cost must be between \$ 92 and \$ 100. Since the option \$ 88 is outside this range, it cannot be the total cost.

Link Problem
Problem 5 Easy

The length and width are both positive integers, and the area is 165. How many distinct rectangular shapes are possible?

  • A.

    2

  • B.

    3

  • C.

    4

  • D.

    5

  • E.

    6

Answer:C

165=1\times 165=3\times 55=5\times 33=11\times 15, there are 4 possibilities.

Link Problem
Problem 6 Easy

As shown in the figure, a cube's 12 edges are painted either red or blue, ensuring that each face has at least one red edge. What is the minimum number of red edges required?

  • A.

    6

  • B.

    5

  • C.

    4

  • D.

    3

  • E.

    2

Answer:D

Select one red-painted edge for length, width, and height, ensuring that the chosen red edges are not in the same plane.

For example, as shown in the figure:

Thus, the correct answer is D.

Link Problem
Problem 7 Easy

To increase the concentration of a 40-kilogram salt solution from 18\% to 20\%, how much water needs to be evaporated?

  • A.

    6 kg

  • B.

    5 kg

  • C.

    4 kg

  • D.

    3 kg

  • E.

    2.5 kg

Answer:C

The amount of salt in the solution remains unchanged before and after evaporation. Let x kilograms of water be evaporated. Then, the equation is:

40 \times 18\% = (40 - x) \times 20\%.

Solving for x, we get:

x = 4.

Thus, the amount of water that needs to be evaporated is \textbf{4 kg}.

Link Problem
Problem 8 Easy

Three cards with the numbers 4, 5, and 6 are available. If these three cards are randomly arranged to form a three-digit number, what is the probability that the resulting number is a multiple of 5?

  • A.

    \frac{1}{6}

  • B.

    \frac{1}{4}

  • C.

    \frac{1}{3}

  • D.

    \frac{1}{2}

  • E.

    \frac{2}{3}

Answer:C

To solve the problem, we first list all possible three-digit numbers that can be formed using the digits 4, 5, and 6: 456, 465, 546, 564, 654, 645. There are 6 such numbers in total. A number is a multiple of 5 if its last digit is 5. From the list, 465 and 645 meet this condition, so there are 2 favorable outcomes. Using the probability formula, the probability of forming a number that is a multiple of 5 is:

\frac{2}{6} = \frac{1}{3}.

Link Problem
Problem 9 Medium

A new operation is defined as:

a \Theta b = \frac{a+b}{2}

Given the equation:

\frac{3}{4} \Theta \left( \frac{1}{6} \Theta \square \right) = \frac{1}{2}

What is the value that should be placed in \square?

  • A.

    3

  • B.

    \frac{1}{3}

  • C.

    \frac{13}{24}

  • D.

    4

  • E.

    \frac{1}{4}

Answer:B

Observing the new operation, it calculates the average of two numbers.

First, treating the parentheses as a whole, the average of the number inside the parentheses and \frac{3}{4} equals \frac{1}{2}, meaning:

\frac{1}{6} \Theta \square = 1 - \frac{3}{4} = \frac{1}{4}.

Thus, the average of \square and \frac{1}{6} equals \frac{1}{4}, leading to:

\square = \frac{1}{4} \times 2 - \frac{1}{6} = \frac{1}{3}.

Thus, the correct answer is B.

Link Problem
Problem 10 Easy

As shown in the figure, each small square has a side length of 2. What is the area of the shaded triangle?

  • A.

    12

  • B.

    16

  • C.

    20

  • D.

    22

  • E.

    24

Answer:D

8\times6=48

2\times6\div2=6

6\times4\div2=12

8\times2\div2=8

48-(6+12+8)=22

Link Problem
Problem 11 Medium

Positive integer n satisfies the equation: 180x = n^2. What is the smallest value of x?

  • A.

    2

  • B.

    5

  • C.

    20

  • D.

    45

  • E.

    180

Answer:B

180=2\times 2\times 3\times 3\times 5={{2}^{2}}\times {{3}^{2}}\times 5, 180x={{n}^{2}}, which is a perfect number, then x=5.

Link Problem
Problem 12 Medium

Students from a middle school are attending a meeting in the auditorium. If each bench seats 3 students, there will be 7 students left standing. However, if each bench seats 4 more students than before, there will be 3 benches left unused. How many students are attending the meeting?

  • A.

    25

  • B.

    26

  • C.

    27

  • D.

    28

  • E.

    29

Answer:D

If each bench seats 4 more students (i.e., 7 students per bench), and there are 3 unused benches, this means there are 21 empty seats in total. The total number of benches is therefore \frac{21 + 7}{4} = 7 \ \text{benches}.

The total number of students is then 7 \times 3 + 7 = 28 \ \text{students}.

Link Problem
Problem 13 Easy

\frac{1}{2\times 3}+\frac{1}{3\times 4}+\frac{1}{4\times 5}+\frac{1}{5\times 6}+\frac{1}{6\times 7}=            .

  • A.

    \frac{1}{2}

  • B.

    \frac{1}{7}

  • C.

    \frac{2}{7}

  • D.

    \frac{5}{14}

  • E.

    \frac{9}{14}

Answer:D

=\left( \frac{1}{2}-\frac{1}{3} \right)+\left( \frac{1}{3}-\frac{1}{4} \right)+\left( \frac{1}{4}-\frac{1}{5} \right)+\left( \frac{1}{5}-\frac{1}{6} \right)+\left( \frac{1}{6}-\frac{1}{7} \right)

=\frac{1}{2}-\frac{1}{3}+\frac{1}{3}-\frac{1}{4}+\frac{1}{4}-\frac{1}{5}+\frac{1}{5}-\frac{1}{6}+\frac{1}{6}-\frac{1}{7}

=\frac{1}{2}-\frac{1}{7}

=\frac{5}{14}

Link Problem
Problem 14 Medium

As shown in the figure, the coordinates of points A and B are (1, 2) and (3, 0), respectively. The triangle \triangle OAB is translated along the positive x-axis so that point B moves to point E, forming the triangle \triangle DCE. If OE = 4, what are the coordinates of point C?

  • A.

    \left(2,2\right)

  • B.

    \left(3,2\right)

  • C.

    \left(1,3\right)

  • D.

    \left(1,4\right)

  • E.

    (3,1)

Answer:A

By analyzing the problem, we know that point B(3, 0) satisfies OB = 3. Since OE = 4, we calculate BE = OE - OB = 1. Therefore, \triangle OAB is translated 1 unit to the right along the x-axis to form \triangle DCE. Point A is also shifted 1 unit to the right, resulting in point C having coordinates (1+1, 2) = (2, 2). Thus, the correct answer is \textbf{A}.

Link Problem
Problem 15 Easy

A three-digit number is divided by 37, resulting in a quotient of a and a remainder of b, where both a and b are non-negative integers. What is the maximum possible value of a + b?

  • A.

    60

  • B.

    62

  • C.

    64

  • D.

    66

  • E.

    68

Answer:B

From the problem statement, the maximum possible value of b is 36.

Since 999 = 37 \times 27

and 998 = 37 \times 26 + 36,

the maximum value of a + b is: 26 + 36 = 62.

Thus, the correct answer is B.

Link Problem
Problem 16 Medium

How many four-digit numbers satisfy the condition that the digit in the units place is greater than the digit in the thousands place, the digit in the thousands place is greater than the digit in the hundreds place, and the digit in the hundreds place is greater than the digit in the tens place?

  • A.

    5040

  • B.

    3024

  • C.

    840

  • D.

    210

  • E.

    105

Answer:D

Since the digits of the four-digit number must follow a strict order (units digit > thousands digit > hundreds digit > tens digit), each selection of 4 distinct digits from 0 to 9 corresponds to exactly one valid number. This is because the order of the digits is uniquely determined by their sizes. Note that 0 can be included in the selection, as it will occupy the smallest position (tens place) if chosen, satisfying the requirement that the leading digit (thousands place) cannot be 0. The total number of ways to choose 4 digits from 0 to 9 is:

\binom{10}{4} = \frac{10 \times 9 \times 8 \times 7}{4 \times 3 \times 2 \times 1} = 210.

Thus, there are 210 such four-digit numbers.

Link Problem
Problem 17 Medium

As shown in the figure, the ancient Greeks often used small pebbles to form various shapes on the sand to study numbers. For example, the numbers 1, 5, 12, 22, \cdots are known as pentagonal numbers. What is the 7-th pentagonal number?

  • A.

    35

  • B.

    51

  • C.

    58

  • D.

    70

  • E.

    72

Answer:D

The sequence of pentagonal numbers is formed as follows:

\text{1st pentagonal number: } 1, \text{2nd pentagonal number: } 1 + 4 = 5, \text{3rd pentagonal number: } 1 + 4 + 7 = 12, \text{4th pentagonal number: } 1 + 4 + 7 + 10 = 22.

Continuing the pattern, we calculate:

\text{5th pentagonal number: } 1 + 4 + 7 + 10 + 13 = 35, \text{6th pentagonal number: } 1 + 4 + 7 + 10 + 13 + 16 = 51, \text{7th pentagonal number: } 1 + 4 + 7 + 10 + 13 + 16 + 19 = 70.

Thus, the 7th pentagonal number is \boxed{70}.

Link Problem
Problem 18 Easy

There is a three-digit number \overline{abc}, the sum of a and c is 10, and \overline{abc} - \overline{bca} = 486. What is the value of a+b-c?

  • A.

    4

  • B.

    5

  • C.

    6

  • D.

    7

  • E.

    None of the above

Answer:C

We can solve this problem using place value .

Step 1: Considering the Units Digit

If no borrowing occurs, we get:

c - a = 6

c + a = 10

Solving these equations:

c = \frac{10 + 6}{2} = 8, \quad a = 8 - 6 = 2.

However, if the minuend’s hundreds digit is 2, the difference cannot be 486, so borrowing must have occurred.

Step 2: Considering Borrowing

With borrowing, we modify the equation:

10 + c - a = 6.

Rearranging:

a - c = 4.

Since a + c = 10, solving for a and c:

a = \frac{10 + 4}{2} = 7, \quad c = 7 - 4 = 3.

Step 3: Finding b

By analyzing the tens and hundreds digits, we determine:

b = 2.

Thus, the three-digit number is 723.

Final Answer:

Thus, the correct answer is 7+2-3=6, which is \boxed{C}.

Link Problem
Problem 19 Medium

Vivi forgot the password to her safe, but she remembers that it consists of four nonzero digits, and the sum of the digits is 9. What is the minimum number of attempts needed to guarantee opening the safe?

  • A.

    52

  • B.

    53

  • C.

    54

  • D.

    55

  • E.

    56

Answer:E

To find all possible passwords, we need to determine combinations of four nonzero digits that sum to 9. The valid combinations are 1, 1, 1, 6; 1, 1, 2, 5; 1, 1, 3, 4; 1, 2, 2, 4; 1, 2, 3, 3; and 2, 2, 2, 3, resulting in six groups. For the combination 1, 1, 1, 6, the digit 6 can occupy any of the 4 positions, giving 4 possibilities. For 1, 1, 2, 5, 2 can occupy 4 positions, 5 can occupy 3 remaining positions, and 1 fills the rest, yielding 4 \times 3 = 12 possibilities. Similarly, each of 1, 1, 3, 4, 1, 2, 2, 4, and 1, 2, 3, 3 also has 12 possibilities. For 2, 2, 2, 3, 3 can occupy any of the 4 positions, giving 4 possibilities. By the addition principle, the total number of passwords is:

4 + 12 + 12 + 12 + 12 + 4 = 56.

Thus, Vivi needs to try at least 56 attempts to guarantee opening the safe.

Link Problem
Problem 20 Easy

As shown in the figure, a grid is formed by squares of side length 1, and 9 grid points (the intersections of the grid lines) are selected. A circle is drawn with center A and radius r. If exactly 3 of the selected points (excluding A) lie inside the circle, what is the range of r?

  • A.

    2\sqrt{2}<{}r<{}\sqrt{17}

  • B.

    \sqrt{17}<{}r\leqslant {}3\sqrt{2}

  • C.

    \sqrt{17}<{}r<{}5

  • D.

    5<{}r<{}\sqrt{29}

  • E.

    5<{}r\leqslant {}3\sqrt{2}

Answer:B

Connect A{{A}_{1}}\sim A{{A}_{8}}

Then, A{{A}_{1}}=\sqrt{{{3}^{2}}+{{3}^{2}}}=3\sqrt{2},

A{{A}_{2}}=\sqrt{{{1}^{2}}+{{4}^{2}}}=\sqrt{17},

A{{A}_{3}}=\sqrt{{{3}^{2}}+{{4}^{2}}}=5,

A{{A}_{4}}=\sqrt{{{5}^{2}}+{{2}^{2}}}=\sqrt{29},

A{{A}_{5}}=\sqrt{{{1}^{2}}+{{4}^{2}}}=\sqrt{17},

A{{A}_{6}}=\sqrt{{{2}^{2}}+{{2}^{2}}}=2\sqrt{2},

A{{A}_{7}}=\sqrt{{{3}^{2}}+{{4}^{2}}}=5,

A{{A}_{8}}=\sqrt{{{3}^{2}}+{{4}^{2}}}=5,

Since 2\sqrt{2}<{}\sqrt{17}<{}3\sqrt{2}<{}5<{}\sqrt{29}.

When \sqrt{17}<{}r\leqslant 3\sqrt{2}, there are exactly 3 points \left( {{A}_{6}},{{A}_{2}},{{A}_{5}} \right) lie inside the circle.

Link Problem
Problem 21 Medium

As shown in the figure, a circular piece of paper is cut into two sectors, \alpha and \beta. The area of \alpha is 18, and the central angle of \alpha is 72^\circ larger than that of \beta. What is the area of \beta?

  • A.

    16

  • B.

    12

  • C.

    14

  • D.

    10

  • E.

    8

Answer:B

Since the two sectors have the same radius, the ratio of their areas is equal to the ratio of their central angles. Given that the difference in their central angles is n_\alpha - n_\beta = 72^\circ and the sum of their central angles is 360^\circ. We can solve for n_\alpha = 216^\circ and n_\beta =144^\circ. Hence, the ratio of their central angles is 216:144 = 3:2. It follows that the area ratio is also 3:2. Thus, the area of \beta is 12.

Link Problem
Problem 22 Medium

What is the number of digits in the product 25^{16} \times 2^{38}?

  • A.

    30

  • B.

    32

  • C.

    34

  • D.

    36

  • E.

    38

Answer:C

The product 25^{16} \times 2^{38} can be simplified as follows 25^{16} \times 2^{38} = 25^{16} \times 4^{19} = (25^{16} \times 4^{16}) \times 4^3.

Since 25 \times 4 = 100, we have 25^{16} \times 4^{16} = (25 \times 4)^{16} = 100^{16}.

Thus, the expression becomes 100^{16} \times 64 = 64 \times 10^{32}.

This represents a number with 2 + 32 = 34 digits (the digits 64 contribute 2 digits, and 10^{32} adds 32 zeros). Therefore, the product 25^{16} \times 2^{38} is a 34-digit number.

Link Problem
Problem 23 Medium

In a school club, \frac{3}{4} of the members participate in community service, and \frac{9}{25} of the members are involved in academic competitions. A total of 44 members participate in both activities. How many members are involved only in academic competitions?

  • A.

    88

  • B.

    90

  • C.

    94

  • D.

    98

  • E.

    100

Answer:E

Let the total number of club members be represented as 1 unit. Using the inclusion-exclusion principle, the total number of members can be calculated as:

\text{Total members} = 44 \div \left( \frac{3}{4} + \frac{9}{25} - 1 \right).

Simplify the expression:

44 \div \left( \frac{111}{100} - 1 \right) = 44 \div \frac{11}{100} = 400 \ \text{members}.

The number of members involved only in academic competitions is:

400 \times \frac{9}{25} - 44 = 144 - 44 = 100.

Thus, the number of members involved only in academic competitions is \boxed{100}.

Link Problem
Problem 24 Easy

There are 6 students, and no two students own the same set of books. However, each pair of students shares exactly one common book, and each book is owned by exactly two students. How many different books are there in total?

  • A.

    12

  • B.

    15

  • C.

    18

  • D.

    21

  • E.

    24

Answer:B

Represent the 6 students as points A_1, A_2, A_3, A_4, A_5, A_6, and use a line connecting two points to indicate that the corresponding pair of students shares exactly one common book. Since each book is owned by exactly two students, there is exactly one line between any two points.

To find the total number of books, we count the total number of lines in a complete graph with 6 points. The number of lines (or edges) in a complete graph with n points is given by:

\binom{n}{2} = \frac{n(n-1)}{2}.

For n = 6, the total number of lines is:

\binom{6}{2} = \frac{6 \times 5}{2} = 15.

Thus, the 6 students share a total of 15 different books. The answer is \boxed{15}.

Link Problem
Problem 25 Easy

A plane can fly for a maximum of 8.5 hours in the air. On its journey, the plane travels at an average speed of 900 \, \text{km/h} going to its destination and 800 \, \text{km/h} on the return trip. What is the greatest distance (in kilometers) the plane can travel one way before it must turn back to ensure it does not exceed its flight time?

  • A.

    3825\text{km}

  • B.

    3400\text{km}

  • C.

    3500

  • D.

    3600\text{km}

  • E.

    3612.5\text{km}

Answer:D

Let the maximum one-way distance the plane can travel be x kilometers. Based on the problem, the equation is

\frac{x}{900} + \frac{x}{800} = 8.5.

Simplifying, we find

8x + 9x = 61200 \quad \Rightarrow \quad 17x = 61200 \quad \Rightarrow \quad x = 3600.

After verification, x = 3600 satisfies the original equation. Therefore, the plane can travel a maximum of 3600 \, \text{km} one way.

Link Problem
Table of Contents
  • 1
  • 2
  • 3
  • 4
  • 5
  • 6
  • 7
  • 8
  • 9
  • 10
  • 11
  • 12
  • 13
  • 14
  • 15
  • 16
  • 17
  • 18
  • 19
  • 20
  • 21
  • 22
  • 23
  • 24
  • 25